17
$\begingroup$

Let us sum a stream of random variables, $X_i \overset{iid}\sim \mathcal{U}(0,1)$; let $Y$ be the number of terms we need for the total to exceed one, i.e. $Y$ is the smallest number such that

$$X_1 + X_2 + \dots + X_Y > 1.$$

Why does the mean of $Y$ equal Euler's constant $e$?

$$\mathbb{E}(Y) = e = \frac{1}{0!} + \frac{1}{1!} + \frac{1}{2!} + \frac{1}{3!} + \dots $$

$\endgroup$
3
  • $\begingroup$ I am posting this in the spirit of a self-study question, though I think I first saw this question over a decade ago. I can't recall how I answered it back then, though I'm sure it wasn't that sprang to mind when I saw this property mentioned in the thread Approximate $e$ using Monte Carlo Simulation. Since I suspect this to be a fairly common exercise question, I have opted to present a sketch rather than a complete solution, though I suppose the main "spoiler warning" belongs in the question itself! $\endgroup$
    – Silverfish
    Feb 6, 2016 at 18:02
  • $\begingroup$ I remain very interested in alternative approaches; I know this was included as a question in Gnedenko's Theory of Probability (originally in Russian but widely translated) but I don't know what solution was expected there, or posed elsewhere. $\endgroup$
    – Silverfish
    Feb 6, 2016 at 20:35
  • 1
    $\begingroup$ I wrote a simulation solution in MATLAB using your simplex method. I didn't know about the link to simplexes, it's so unexpected. $\endgroup$
    – Aksakal
    Feb 7, 2016 at 3:33

3 Answers 3

17
$\begingroup$

First observation: $Y$ has a more pleasing CDF than PMF

The probability mass function $p_Y(n)$ is the probability that $n$ is "only just enough" for the total to exceed unity, i.e. $X_1 + X_2 + \dots X_n$ exceeds one while $X_1 + \dots + X_{n-1}$ does not.

The cumulative distribution $F_Y(n) = \Pr(Y \leq n)$ simply requires $n$ is "enough", i.e. $\sum_{i=1}^{n}X_i > 1$ with no restriction on how much by. This looks like a much simpler event to deal with the probability of.

Second observation: $Y$ takes non-negative integer values so $\mathbb{E}(Y)$ can be written in terms of the CDF

Clearly $Y$ can only take values in $\{0, 1, 2, \dots\}$, so we can write its mean in terms of the complementary CDF, $\bar F_Y$.

$$\mathbb{E}(Y) = \sum_{n=0}^\infty \bar F_Y(n) = \sum_{n=0}^\infty \left(1 - F_Y(n) \right)$$

In fact $\Pr(Y=0)$ and $\Pr(Y=1)$ are both zero, so the first two terms are $\mathbb{E}(Y) = 1 + 1 + \dots$.

As for the later terms, if $F_Y(n)$ is the probability that $\sum_{i=1}^{n}X_i > 1$, what event is $\bar F_Y(n)$ the probability of?

Third observation: the (hyper)volume of an $n$-simplex is $\frac{1}{n!}$

The $n$-simplex I have in mind occupies the volume under a standard unit $(n-1)$-simplex in the all-positive orthant of $\mathbb{R}^n$: it is the convex hull of $(n+1)$ vertices, in particular the origin plus the vertices of the unit $(n-1)$-simplex at $(1, 0, 0, \dots)$, $(0, 1, 0, \dots)$ etc.

volumes of 2-simplex and 3-simplex

For example, the 2-simplex above with $x_1 + x_2 \leq 1$ has area $\frac{1}{2}$ and the 3-simplex with $x_1 + x_2 + x_3 \leq 1$ has volume $\frac{1}{6}$.

For a proof that proceeds by directly evaluating an integral for the probability of the event described by $\bar F_Y(n)$, and links to two other arguments, see this Math SE thread. The related thread may also be of interest: Is there a relationship between $e$ and the sum of $n$-simplexes volumes?

$\endgroup$
3
  • 1
    $\begingroup$ This is an interesting geometric approach, and easy to solve this way. Beautiful. Here's the equation for a volume of a simplex. I don't think there could be a more elegant solution, frankly $\endgroup$
    – Aksakal
    Feb 6, 2016 at 19:05
  • 1
    $\begingroup$ +1 You can also obtain the full distribution of $Y$ from any of the approaches in my post at stats.stackexchange.com/questions/41467/…. $\endgroup$
    – whuber
    Feb 6, 2016 at 19:20
  • $\begingroup$ If I stumbled on this solution, there's no way they could force me do it other way in a school :) $\endgroup$
    – Aksakal
    Feb 7, 2016 at 3:38
11
$\begingroup$

Fix $n \ge 1$. Let $$U_i = X_1 + X_2 + \cdots + X_i \mod 1$$ be the fractional parts of the partial sums for $i=1,2,\ldots, n$. The independent uniformity of $X_1$ and $X_{i+1}$ guarantee that $U_{i+1}$ is just as likely to exceed $U_i$ as it is to be less than it. This implies that all $n!$ orderings of the sequence $(U_i)$ are equally likely.

Given the sequence $U_1, U_2, \ldots, U_n$, we can recover the sequence $X_1, X_2, \ldots, X_n$. To see how, notice that

  • $U_1 = X_1$ because both are between $0$ and $1$.

  • If $U_{i+1} \ge U_i$, then $X_{i+1} = U_{i+1} - U_i$.

  • Otherwise, $U_i + X_{i+1} \gt 1$, whence $X_{i+1} = U_{i+1} - U_i + 1$.

There is exactly one sequence in which the $U_i$ are already in increasing order, in which case $1 \gt U_n = X_1 + X_2 + \cdots + X_n$. Being one of $n!$ equally likely sequences, this has a chance $1/n!$ of occurring. In all the other sequences at least one step from $U_i$ to $U_{i+1}$ is out of order. This implies the sum of the $X_i$ had to equal or exceed $1$. Thus we see that

$$\Pr(Y \gt n) = \Pr(X_1 + X_2 + \cdots + X_n \le 1) = \Pr(X_1 + X_2 + \cdots + X_n \lt 1) = \frac{1}{n!}.$$

This yields the probabilities for the entire distribution of $Y$, since for integral $n\ge 1$

$$\Pr(Y = n) = \Pr(Y \gt n-1) - \Pr(Y \gt n) = \frac{1}{(n-1)!} - \frac{1}{n!} = \frac{n-1}{n!}.$$

Moreover,

$$\mathbb{E}(Y) = \sum_{n=0}^\infty \Pr(Y \gt n) = \sum_{n=0}^\infty \frac{1}{n!} = e,$$

QED.

$\endgroup$
8
  • $\begingroup$ I have read it a couple of times, and I almost get it... I posted a couple of questions in the Mathematics SE as a result of the $e$ constant computer simulation. I don't know if you saw them. One of them came back before your kind explanation on Tenfold about the ceiling function of the $1/U(0,1)$ and the Taylor series. The second one was exactly about this topic, never got a response, until now... $\endgroup$ Feb 7, 2016 at 2:04
  • $\begingroup$ here and here. $\endgroup$ Feb 7, 2016 at 4:52
  • $\begingroup$ And could you add the proof with the uniform spacings as well? $\endgroup$
    – Xi'an
    Feb 8, 2016 at 12:15
  • $\begingroup$ @Xi'an Could you indicate more specifically what you mean by "uniform spacings" in this context? $\endgroup$
    – whuber
    Feb 8, 2016 at 13:21
  • 1
    $\begingroup$ @simran There are $n!$ ways of ordering $n$ numbers; if there are no ties (a tie has probability $0$ here) then just one has them in increasing order. $\Pr(Y>n)=\frac 1{n!}$ is the probability of not having exceeded $1$ in the first $n$ attempts so gives the complementary CDF. $\endgroup$
    – Henry
    Mar 12 at 1:51
7
$\begingroup$

In Sheldon Ross' A First Course in Probability there is an easy to follow proof:

Modifying a bit the notation in the OP, $U_i \overset{iid}\sim \mathcal{U}(0,1)$ and $Y$ the minimum number of terms for $U_1 + U_2 + \dots + U_Y > 1$, or expressed differently:

$$Y = min\Big\{n: \sum_{i=1}^n U_i>1\Big\}$$

If instead we looked for:

$$Y(u) = min\Big\{n: \sum_{i=1}^n U_i>u\Big\}$$ for $u\in[0,1]$, we define the $f(u)=\mathbb E[Y(u)]$, expressing the expectation for the number of realizations of uniform draws that will exceed $u$ when added.

We can apply the following general properties for continuous variables:

$E[X] = E[E[X|Y]]=\displaystyle\int_{-\infty}^{\infty}E[X|Y=y]\,f_Y(y)\,dy$

to express $f(u)$ conditionally on the outcome of the first uniform, and getting a manageable equation thanks to the pdf of $X \sim U(0,1)$, $f_Y(y)=1.$ This would be it:

$$f(u)=\displaystyle\int_0^1 \mathbb E[Y(u)|U_1=x]\,dx \tag 1$$

If the $U_1=x$ we are conditioning on is greater than $u$, i.e. $x>u$, $\mathbb E[Y(u)|U_1=x] =1 .$ If, on the other hand, $x <u$, $\mathbb E[Y(u)|U_1=x] =1 + f(u - x)$, because we already have drawn $1$ uniform random, and we still have the difference between $x$ and $u$ to cover. Going back to equation (1):

$$f(u) = 1 + \displaystyle\int_0^x f(u - x) \,dx$$, and with substituting $w = u - x$ we would have $f(u) = 1 + \displaystyle\int_0^x f(w) \,dw$.

If we differentiate both sides of this equation, we can see that:

$$f'(u) = f(u)\implies \frac{f'(u)}{f(u)}=1$$

with one last integration we get:

$$log[f(u)] = u + c \implies f(u) = k \,e^u$$

We know that the expectation that drawing a sample from the uniform distribution and surpassing $0$ is $1$, or $f(0) = 1$. Hence, $k = 1$, and $f(u)=e^u$. Therefore $f(1) = e.$

$\endgroup$
1
  • 1
    $\begingroup$ I do like the manner in which this generalises the result. $\endgroup$
    – Silverfish
    Feb 7, 2016 at 9:50

Your Answer

By clicking “Post Your Answer”, you agree to our terms of service and acknowledge you have read our privacy policy.

Not the answer you're looking for? Browse other questions tagged or ask your own question.